\( \newcommand{\N}{\mathbb{N}} \newcommand{\R}{\mathbb{R}} \newcommand{\C}{\mathbb{C}} \newcommand{\Q}{\mathbb{Q}} \newcommand{\Z}{\mathbb{Z}} \newcommand{\P}{\mathcal P} \newcommand{\B}{\mathcal B} \newcommand{\F}{\mathbb{F}} \newcommand{\E}{\mathcal E} \newcommand{\brac}[1]{\left(#1\right)} \newcommand{\abs}[1]{\left|#1\right|} \newcommand{\matrixx}[1]{\begin{bmatrix}#1\end {bmatrix}} \newcommand{\vmatrixx}[1]{\begin{vmatrix} #1\end{vmatrix}} \newcommand{\lims}{\mathop{\overline{\lim}}} \newcommand{\limi}{\mathop{\underline{\lim}}} \newcommand{\limn}{\lim_{n\to\infty}} \newcommand{\limsn}{\lims_{n\to\infty}} \newcommand{\limin}{\limi_{n\to\infty}} \newcommand{\nul}{\mathop{\mathrm{Nul}}} \newcommand{\col}{\mathop{\mathrm{Col}}} \newcommand{\rank}{\mathop{\mathrm{Rank}}} \newcommand{\dis}{\displaystyle} \newcommand{\spann}{\mathop{\mathrm{span}}} \newcommand{\range}{\mathop{\mathrm{range}}} \newcommand{\inner}[1]{\langle #1 \rangle} \newcommand{\innerr}[1]{\left\langle #1 \right \rangle} \newcommand{\ol}[1]{\overline{#1}} \newcommand{\toto}{\rightrightarrows} \newcommand{\upto}{\nearrow} \newcommand{\downto}{\searrow} \newcommand{\qed}{\quad \blacksquare} \newcommand{\tr}{\mathop{\mathrm{tr}}} \newcommand{\bm}{\boldsymbol} \newcommand{\cupp}{\bigcup} \newcommand{\capp}{\bigcap} \newcommand{\sqcupp}{\bigsqcup} \newcommand{\re}{\mathop{\mathrm{Re}}} \newcommand{\im}{\mathop{\mathrm{Im}}} \newcommand{\comma}{\text{,}} \newcommand{\foot}{\text{。}} \)

Tuesday, March 30, 2010

出左分, 一些可介紹給中四的不等式。

MATH190 20/20
MATH151 100/100


在中四的數學科 (DSE 微分應該必修?不清楚) 裏,談到不等式,最多只可以問一些 $ |\sin x|, |\cos x|\leq 1$ 這類型 fact 或 $ f(x)=ax^2+bx+c$ 可以有最大/最少值的問題。

現在問,若要求你找 (例如) $\displaystyle \frac{\sin \phi}{2-\cos\phi}$ 在 $ 0\leq \phi\leq \frac{\pi}{2}$ 的最大值,作為一個中四沒有 a.maths 的學生,怎樣找?這是一個可以考考學生的問題。

Monday, March 29, 2010

開始憎 multivariable integration

唉!星期 2 仲要頂兩堂 204 lecture,聽日內要明哂第一堂 lecture 講的野,有排受。

Saturday, March 27, 2010

Simple revision on differentiation

Problem 1. Suppose $  f(1) = f(2) = 0$, $  f(3) = 1$ and $  f$ is twice differentiable on $  [0,3]$. Show that $  f''(c)>\frac{1}{2},\exists c\in(0,3)$.

Problem 2. Suppose $  f(0) = 0, f(1) = 1$, $  f$ is differentiable on $  [0,1]$. Show that $  \displaystyle \frac{1}{f'(a)}+\frac{1}{f'(b)}=2$, for some distinct $  a,b\in (0,1)$.

兩條都是從某中學教師的 BLOG 中抽出來,後一條加上 distinct 的原因是為了把難度增加 (從 BLOG 中抽出來時沒有加上 distinct)。若把 distinct 移走,那很明顯存在 $  f'(c)=1$, 取 $  a=b=c$ 便完成證明。

Thursday, March 25, 2010

重新再想 Lagrange's multiplier

學了多元微分學後重新再想及證明 Lagrange's multipler,有錯處的話請提省我!
http://ihome.ust.hk/~cclee/document/analysiswaytothink.pdf

利用相同的證法裏面的思想可推廣至任何一般情形,基本上我是閱讀了那個一般情形的證明再自行嘗試證明這個較常用的 case。

讀者有 math301 背景比較容易利解。
需要知識:Multivariable differentiation, implicit function theorem.

Wednesday, March 24, 2010

Just a review of Lagrange's Multiplier (to be corrected)

We try to create a concrete computation to find the local extreme of $  f:\mathbb{R}^n\supset U\to\mathbb{R}$ subject to the constraint $  g(\vec{x})=c$. Here $  c$ is a regular point, namely, a point such that $  g(\vec{x})=c\implies \nabla g(\vec x)\neq \vec{0}$.

Moreover, we call the surface specified by $  g(\vec x) = c$ to be $  S$, hence we are finding the local extreme of $  f$ restricted to the surface $  S$, namely, we are finding the local extreme of $  \big.f\big|_S$. Suppose now $  \big.f\big|_S$ attains the local extreme at $  \vec x = \vec x_0$, then for any $  \vec v\in\{\vec u\in\mathbb{R}^n:\nabla g(\vec x_0)\cdot \vec u = \vec 0\}:=T_{\vec x_0}S$, i.e. any tangent vector moving on $  S$ at $  \vec x_0$, we have $  D_{\vec v}f(\vec x_0)=\nabla f(\vec x_0)\cdot \vec v=0$ (for otherwise it is not an extrema).

Let's summarize the implication, $  \forall \vec v\in T_{\vec x_0}S\implies \nabla f(\vec x_0)\cdot \vec v=0$.

Since $  \nabla g(\vec x_0)$ spans the normal space of $  S$ at $  \vec x_0$, it follows that $  \nabla f(\vec x_0)=\lambda \nabla g(\vec x_0)$.

Monday, March 22, 2010

ODE

有陣時 integrator factor 真係 ``observe" 出黎,冇既定 procedure =.=。

$  \boxed{\text{\textbf{\sffamily Problem}}}$
Find an appropriate integrating factor and solve $  (x^3y^2-y)\,\mathrm dx+(x^2y^4-x)\,\mathrm dy=0$

为什么要用 (1-5)

Sunday, March 21, 2010

屋企真係一個寶庫

無喇喇搵到一本關於 ODE ge 書 (學校 textbook 太貴買唔起),內有 500 多條 solved problem = =,又有 brief review of 相關 material,正好補返我冇上堂聽 ge 不足,睇 powerpoint 真係睇到好攰 (我一直都係電腦睇)。

今日返學校去拎返份 math 190 功課,TA 語重心長咁俾左一個 comment 我。
645644
在 latex 裏打 $\sum$ 會有 $\sum$,打 $\displaystyle \sum$ 會有 $\displaystyle \sum$,這就是 displaystyle。

Problem. Let  $ f:[0,\infty)\to\mathbb{R}$ with $ f(0)=-1$ be a differentiable function so that $ |f(x)-f'(x)|<1,\forall x\ge 0$.

a) Prove that $ f$ does have a limit that is infinite.
b) Give an example of such a function.

Problem. Evaluate $\displaystyle\lim_{n\to\infty }n\left(\frac{1^{\alpha }+2^{\alpha }+...+n^{\alpha }}{n^{\alpha+1 }}-\frac{1}{\alpha+1 }\right)$, $ \alpha > 1 $.

Saturday, March 20, 2010

陳志雲陳水扁有「真的假不了,假的真不了」

我有「好的說不了,說的好不了。」
我的嘴蠻臭。

最近在高登看到不錯的不等式題目,當然你知道 Jensen's inequality 的話可以把這三條秒殺。
現在限制自己只知道 Cauchy-Schwarz inequality,考考腦筯,試證明:
對於正數 $  x_1,x_2,\dots,x_n$, $  m,n\in\mathbb{N}$,有

(a) $  \displaystyle \frac{\sum_{i=1}^nx_i}{n}\leq\left(\frac{\sum_{i=1}^nx_i^2}{n}\right)^{1/2}\leq \left(\frac{\sum_{i=1}^nx_i^3}{n}\right)^{1/3}$
更一般地,證明
(b) $  \displaystyle \frac{\sum_{i=1}^nx_i}{n}\leq\left(\frac{\sum_{i=1}^nx_i^m}{n}\right)^{1/m}$.

這裏所謂的一般跟你所期望的可能有點不同,事實上,一般地對於正整數 $  m$ 以下的不等式依然成立,\[
\left(\frac{\sum_{i=1}^nx_i^m}{n}\right)^{1/m}\leq \left(\frac{\sum_{i=1}^nx_i^{m+1}}{n}\right)^{1/{(m+1)}}.
\]

容易由 Jensen's inequality 得到結論,well... 我不確定 Cauchy 是否也能夠成功用在它的證明上 (我相信它不是萬能吧.....),有興趣的可以試試,做到的話記得通知我,我會放到我的 problem book 裏。
(c) 對於 $  \beta_1,\beta_2,\dots,\beta_n\in\mathbb{Q}^+$,$  \beta = \beta_1+\beta_2+\cdots+\beta_n$,有 \[
\frac{\sum_{i=1}^n\beta_ix_i}{\beta}\leq \left(\frac{\sum_{i=1}^n\beta_ix_i^m}{\beta}\right)^{1/m}.
\]
現在 MATH202 正在教一些有關「測度」的知識。它令我解決了 4 個多月來的疑惑。
從前嚴民教授提到:「An integrable function must be continuous somewhere」,嚴民教授只是順便講講這個 fact,我弄不明白,便走去問了 LCM。但最後是不了了之 (應該是考慮到我不懂 Lebesgue's theorem 吧)。有了這個定理要證明它真係十分簡單,假設那個 function 處處不連續,那麼它的所有斷點所集成的一個集並不是一個零測集 (of measure zero),也就是說那 function 本身不是 integrable 的,矛盾。因此,一個 integrable function 必定在某處連續。有了這個 fact,又可以證其他 fact 了。

Sunday, March 14, 2010

Problem. Verify that $  z=z(x,y)$ which is implicitly defined by $  \displaystyle x^2+y^2+z^2=yf\left(\frac{z}{y}\right)$ satisfies the partial diiferential equation
$  \displaystyle (x^2-y^2-z^2)\frac{\partial z}{\partial x}+2xy\frac{\partial z}{\partial y}=4xz$.

要證 2 個 statement 等價,只須證明 $  (1)\implies (2)\implies (1)$, 但去到 4, 5 或更多個 statement,順序證明有可能令我們讚進死胡同。打個比方,若要證明 4 個 statement 等價,先證 $  (1)\implies (2)\implies (3)$ 卻發現自己 $  (3)\implies (4)$ 怎樣也想不到,可是 $  (2)\implies (4)$ 卻十分簡單,不妨先完成 $  (1)\implies (2)\implies (3)\implies (1)$,再從中把 $  (2)$ 抽出來證明 $  (2)\iff (4)$。這裏我們需要多做一步,但相對地可把問題變得簡單。

Math202 那份關於 integrability 的 notes,最後的第二條,有關證明四個命題等價的問題,若發現由第 3 到第 4 出現困難的話,不妨蹺一條長一點,但較平坦的路。

最後,當大家大至上認為自己對 integrability 有一定的認識,可嘗試 09 spring math203 final 有關 integrability 的題目:

Problem. Suppose $  f(x)$ and $  g(x)$ are integrable on $  [a,b]$. Prove that for any $  \epsilon > 0$, there is $  \delta>0$, such that for any partition $  P$ satisfying $  \|P\|<\delta$ and choices $  x_i^*,x_i^{**}\in [x_{i-1},x_i]$, we have \[
\left|\sum f(x_i^*)g(x_i^{**})\Delta x_i-\int_a^bf(x)g(x)\,\mathrm{d} x\right|<\epsilon.
\] 這大至上證明了,就算 $  x_i^*$ 和 $  x_i^{**}$ 所取的值不同,同樣有和相同選擇時的結果。

Friday, March 12, 2010

Tuesday, March 9, 2010

科大獸醫

一位我們學校某學生的經歷 = =。


###!!!@@@ceev 說: (18小時前)
我比科大獸醫玩死

###!!!@@@@ceev 說: (18小時前)
上個星期去看獸醫 諗住傷風 easy job啦
開左三日藥比我 點知食完之後無好之餘 仲要大獲左
今日頂唔順去看醫生 點知話個鼻發炎
真係吹漲 獸醫可以完全唔知我個鼻發炎 我一早d鼻涕就green

Sunday, March 7, 2010

把 pdf 合併

同學以前問我懂不懂把數個 pdf 合併,嗯... 我不懂。今天心血來潮在網上找找,比較易找的都是 freeware,要錢的。用 ``Merge pdf" 找找看,發現一個不錯的網頁 (link)

不用下載任何工具,上傳數個想要合併的 pdf,按 merge,完成!最後下載回來,方便快捷簡單易用。

Problem. 設函數 $f$ 在 $latex x=0$ 處連續,如果 $ \displaystyle \lim_{x\to 0}\frac{f(2x)-f(x)}{x}=m$,求證 $ f'(0)=m$.

做這題時可能還須要用到以下結論。

Prerequisite. 設 $ \lim_{x\to 0}f(x)=0$,且 $ \displaystyle f(x)-f\left(\frac{x}{2}\right)=o(x)$ ($x\to 0$),求證:$\displaystyle f(x)=o(x)$ ($latex x\to 0$)。

Friday, March 5, 2010

Well...

睇黎我要背鬼左呢兩條 inequality 佢=.=,起碼要由右手邊 sense 到左手邊舊野。
  • $  a^3+b^3+c^3+3abc \geq a^2 b+a^2 c+b^2 a+b^2 c+c^2 a+c^2 b$.
  • $  a^4+b^4+c^4+abc(a+b+c) \geq a^3 b+a^3 c+b^3 a+b^3 c+c^3 a+c^3 b$.

Wednesday, March 3, 2010

熱死人

呢幾日都好翳焗,熱死人了……。

看着嚴民教授那 inverse function theorem 的三頁證明,目前還抱着監賞的目光。

Monday, March 1, 2010

Problems of MATH202 # 17 of the last week

Since I am busy doing my work in another course, this weekly post is delayed til today :(.
Of the problems, only 5, 6, 7 are of our interest.

Problem 5. Let $  f:\mathbb{R}\to\mathbb{R}$, be a three times differentiable function. If $  f(x)$ and $  f'''(x)$ are bounded functions on $  \mathbb{R}$, show that $  f'$ and $  f''$ are also bounded functions on $  \mathbb{R}$.

My way.
Just make good use of the expansion $  f(x+h)=f(x)+f'(x)h+\frac{1}{2}f''(x)h^2+\frac{1}{3!}f'''(x+\theta h)h^3$, for some $  \theta \in (0,1)$.

Problem 6. If $  f(x)$ and $  g(x)$ are $  n$ times differentiable and $  f^{(n-1)}(x),g^{(n-1)}(x)$ are both continuous in $  [a,b$. Then there exists a number $  c \in(a,b)$ such that \[ \frac{\displaystyle f(b)-f(a)-\sum_{k=1}^{n-1}\frac{(b-a)^k}{k!}f^{(k)}(a)}{\displaystyle g(b)-g(a)-\sum_{k=1}^{m-1}\frac{(b-a)^k}{k!}g^{(k)}(a)}=\frac{(m-1)!}{(n-1)!}(b-c)^{n-m}\left(\frac{f^{(n)}(c)}{g^{(m)}(c)}\right).\]

NO IDEA, I am just able to prove the case when $  m=n$.

Problem 7. Let $  f$ be $  p$ times differentiable on $  \mathbb{R}$ and let $  M_k=\sup\{|f^{(k)}(x)|:x\in\mathbb{R}\}<\infty$, $  k=0,1,2,\dots,p$ and $  p\ge 2$. Prove that $  M_1\leq \sqrt{2M_0M_2}$ and \[M_k\leq 2^{\frac{k(p-k)}{2}}M_0^{1-\frac{k}{p}}M_p^{\frac{k}{p}}$, for $  k=1,2,\dots,p-1.\]
My way.
(1) Same as the case of problem 5, replace $  h$ by $  -h$ to construct another equation (remember to choose different $  \theta$), subtract two equation, observe that $  0\leq 2M_0 +2M_1h+M_2h^2$ for any $  h$, while discriminant $  \leq 0$, we are done.
(2) In exactly the same manner as (1), we conclude that $  M_{j+1}\leq \sqrt{2M_jM_{j+2}}$ for all $  j\leq p-2$. Now we take product $  \prod_{j=m}^{n}$ on both sides, having \[ \sqrt{M_{n+1}M_{m+1}}\leq (\sqrt{2})^{n-m+1}\sqrt{M_{n+2}M_m}.
\] Before we proceed, we first consider two cases. If $  M_k=0$, then the inequality we are asked to prove obviously holds since right hand side is always non-negative. In case if $  M_k>0$, then we take the product $  \prod_{m=0}^{k-1}$ on both sides of $  \sqrt{M_{n+1}M_{m+1}}\leq (\sqrt{2})^{n-m+1}\sqrt{M_{n+2}M_m}$, it results in \[ M_n^k\leq \left(\frac{M_0}{M_k}2^{k(2n-k+1)/2}\right)M_{n+1}^k.\]
(I remember I have replaced $  n$ by $  n-1$ to make the inequality seem better), we are interested in this because it is a beautiful (in the sense of solving the problem) reccurence relation, we have a direct consequence \begin{align*}
 M_n^k&\leq \left(\frac{M_0}{M_k}\right)^{p-k}2^{\sum_{j=n}^{n+p-k-1}k(2j-k+1)/2}M_{n+p-k}^k\\
&=\left(\frac{M_0}{M_k}\right)^{p-k}2^{k(p-k)(2n-2k+p)/2}M_{n+p-k}^k.
\end{align*} Finally, we take $  n=k$, $  M_k^p\leq M_0^{p-k}2^{\frac{k(p-k)p}{2}}M_p^{k}$.